Hypercube and Hyperspheres












16












$begingroup$


Let $n,kinmathbb{N}$. In this problem, the geometry of $mathbb{R}^n$ is the usual Euclidean geometry.




The lattice hypercube $ Q(n,k)$ is defined to be the set $ {1,2,...,k}^n subseteqmathbb{R}^n$. If we want to cover this hypercube with the $m$
distinct $(n-1)$-dimensional hyperspheres $S_1$, $S_2$, $ldots$,
$S_m$. By "covering $Q(n,k)$," I mean that $Q(n,k)subseteq
bigcuplimits_{j=1}^m,S_j$
. What is the least possible value of $ m$?




Using a simple Combinatorial Nullstellensatz argument, we can show that if $ mu(n,k)$ is the minimum possible $ m$, then
$$ 1 + nleftlfloordfrac{k - 1}{2}rightrfloor leq mu(n,k) leq 1 + nleftlfloorleft(dfrac{k - 1}{2}right)^2rightrfloor ,.
$$

However, the lower bound is not sharp for large $ k$. All I know is that the lower bound is exact for $ k = 1$, $ 2$, $ 3$, and $ 4$, as well as when $ n = 1$, and for a special case with $ n = 2$ and $ k = 5$ (as shown in the attached figure below). While my bounds give $5leq mu(2,6) leq 13$, I believe that $mu(2,6)=6$. Can you verify this? enter image description here



Can anyone help improve these bounds? Note that the bounds also work if the $S_j$'s can be $(n-1)$-dimensional hyperellipsoids, but what will be the value of the smallest $m$ in this case (which, to avoid confusion, we shall use $nu(n,k)$ for this smallest $m$)? In other words, we also have
$$ 1 + nleftlfloordfrac{k - 1}{2}rightrfloor leq nu(n,k) leq 1 + nleftlfloorleft(dfrac{k - 1}{2}right)^2rightrfloor ,.
$$


For example, $nu(1,k)=leftlceildfrac{k+1}{2}rightrceil=mu(1,k)$ for all $kinmathbb{N}$ and $nu(2,k)=1+2leftlfloordfrac{k-1}{2}rightrfloor$ for $k=1,2,3,4,5$. Observe that the lower bound for $nu(2,6)$ is also sharp (i.e., $nu(2,6)=5$).



What happens if the $S_j$'s can be any nondegenerate $(n-1)$-dimensional hyperconic sections (where my Combinatorial Nullstellensatz argument no longer works, so the lower bound I have obtained no longer holds)? In this most general form, we shall use the notation $rho(n,k)$ for the smallest $m$. The only known bound for $rho(n,k)$ is
$$ rho(n,k) leq 1 + nleftlfloorleft(dfrac{k - 1}{2}right)^2rightrfloor ,.
$$

For example, $rho(1,k)=leftlceildfrac{k+1}{2}rightrceil=mu(1,k)$ for all $kinmathbb{N}$. However, $mu$, $nu$, and $rho$ do not always coincide. Known values are $rho(2,1)=1=rho(2,2)$ and $rho(2,3)=2<mu(2,3)$, while we have $rho(2,4)leq 3=mu(2,4)$, $rho(2,5)leq 4 <mu(2,5)$, and $rho(2,6)leq 5leqmu(2,6)$.










share|cite|improve this question











$endgroup$












  • $begingroup$
    According to your question, 2d circles should be used in a 3d lattice hypercube. And 1d line-segments should be used in a 2d lattice hypercube. So why do you use 2d circles in the 2d lattice hypercube?
    $endgroup$
    – johannesvalks
    Jul 27 '15 at 4:47










  • $begingroup$
    The usual definition is that a unit $n$-dimensional sphere is the set $mathbb{S}^n = left{mathbf{x}inmathbb{R}^{n+1},|,|mathbf{x}|_2=1right}$, which sits in the $(n+1)$-dimensional Euclidean space. Hence, a circle is a $1$-dimensional sphere. This is consistent with the notion of manifold dimensions.
    $endgroup$
    – Batominovski
    Jul 27 '15 at 5:01












  • $begingroup$
    Ok, thanks. Now I can go to this problem again.
    $endgroup$
    – johannesvalks
    Jul 27 '15 at 5:03






  • 2




    $begingroup$
    Various papers I found that may or may not be helpful: uam.es/personal_pdi/ciencias/fchamizo/kiosco/files/lpha.pdf, link.springer.com/article/10.1007%2Fs006050050066, digital.csic.es/bitstream/10261/31107/1/… (Theorem 15), digital.csic.es/bitstream/10261/31179/1/…, uam.es/personal_pdi/ciencias/cillerue/Papers/closelattice.pdf.
    $endgroup$
    – joriki
    Jul 29 '15 at 14:41
















16












$begingroup$


Let $n,kinmathbb{N}$. In this problem, the geometry of $mathbb{R}^n$ is the usual Euclidean geometry.




The lattice hypercube $ Q(n,k)$ is defined to be the set $ {1,2,...,k}^n subseteqmathbb{R}^n$. If we want to cover this hypercube with the $m$
distinct $(n-1)$-dimensional hyperspheres $S_1$, $S_2$, $ldots$,
$S_m$. By "covering $Q(n,k)$," I mean that $Q(n,k)subseteq
bigcuplimits_{j=1}^m,S_j$
. What is the least possible value of $ m$?




Using a simple Combinatorial Nullstellensatz argument, we can show that if $ mu(n,k)$ is the minimum possible $ m$, then
$$ 1 + nleftlfloordfrac{k - 1}{2}rightrfloor leq mu(n,k) leq 1 + nleftlfloorleft(dfrac{k - 1}{2}right)^2rightrfloor ,.
$$

However, the lower bound is not sharp for large $ k$. All I know is that the lower bound is exact for $ k = 1$, $ 2$, $ 3$, and $ 4$, as well as when $ n = 1$, and for a special case with $ n = 2$ and $ k = 5$ (as shown in the attached figure below). While my bounds give $5leq mu(2,6) leq 13$, I believe that $mu(2,6)=6$. Can you verify this? enter image description here



Can anyone help improve these bounds? Note that the bounds also work if the $S_j$'s can be $(n-1)$-dimensional hyperellipsoids, but what will be the value of the smallest $m$ in this case (which, to avoid confusion, we shall use $nu(n,k)$ for this smallest $m$)? In other words, we also have
$$ 1 + nleftlfloordfrac{k - 1}{2}rightrfloor leq nu(n,k) leq 1 + nleftlfloorleft(dfrac{k - 1}{2}right)^2rightrfloor ,.
$$


For example, $nu(1,k)=leftlceildfrac{k+1}{2}rightrceil=mu(1,k)$ for all $kinmathbb{N}$ and $nu(2,k)=1+2leftlfloordfrac{k-1}{2}rightrfloor$ for $k=1,2,3,4,5$. Observe that the lower bound for $nu(2,6)$ is also sharp (i.e., $nu(2,6)=5$).



What happens if the $S_j$'s can be any nondegenerate $(n-1)$-dimensional hyperconic sections (where my Combinatorial Nullstellensatz argument no longer works, so the lower bound I have obtained no longer holds)? In this most general form, we shall use the notation $rho(n,k)$ for the smallest $m$. The only known bound for $rho(n,k)$ is
$$ rho(n,k) leq 1 + nleftlfloorleft(dfrac{k - 1}{2}right)^2rightrfloor ,.
$$

For example, $rho(1,k)=leftlceildfrac{k+1}{2}rightrceil=mu(1,k)$ for all $kinmathbb{N}$. However, $mu$, $nu$, and $rho$ do not always coincide. Known values are $rho(2,1)=1=rho(2,2)$ and $rho(2,3)=2<mu(2,3)$, while we have $rho(2,4)leq 3=mu(2,4)$, $rho(2,5)leq 4 <mu(2,5)$, and $rho(2,6)leq 5leqmu(2,6)$.










share|cite|improve this question











$endgroup$












  • $begingroup$
    According to your question, 2d circles should be used in a 3d lattice hypercube. And 1d line-segments should be used in a 2d lattice hypercube. So why do you use 2d circles in the 2d lattice hypercube?
    $endgroup$
    – johannesvalks
    Jul 27 '15 at 4:47










  • $begingroup$
    The usual definition is that a unit $n$-dimensional sphere is the set $mathbb{S}^n = left{mathbf{x}inmathbb{R}^{n+1},|,|mathbf{x}|_2=1right}$, which sits in the $(n+1)$-dimensional Euclidean space. Hence, a circle is a $1$-dimensional sphere. This is consistent with the notion of manifold dimensions.
    $endgroup$
    – Batominovski
    Jul 27 '15 at 5:01












  • $begingroup$
    Ok, thanks. Now I can go to this problem again.
    $endgroup$
    – johannesvalks
    Jul 27 '15 at 5:03






  • 2




    $begingroup$
    Various papers I found that may or may not be helpful: uam.es/personal_pdi/ciencias/fchamizo/kiosco/files/lpha.pdf, link.springer.com/article/10.1007%2Fs006050050066, digital.csic.es/bitstream/10261/31107/1/… (Theorem 15), digital.csic.es/bitstream/10261/31179/1/…, uam.es/personal_pdi/ciencias/cillerue/Papers/closelattice.pdf.
    $endgroup$
    – joriki
    Jul 29 '15 at 14:41














16












16








16


2



$begingroup$


Let $n,kinmathbb{N}$. In this problem, the geometry of $mathbb{R}^n$ is the usual Euclidean geometry.




The lattice hypercube $ Q(n,k)$ is defined to be the set $ {1,2,...,k}^n subseteqmathbb{R}^n$. If we want to cover this hypercube with the $m$
distinct $(n-1)$-dimensional hyperspheres $S_1$, $S_2$, $ldots$,
$S_m$. By "covering $Q(n,k)$," I mean that $Q(n,k)subseteq
bigcuplimits_{j=1}^m,S_j$
. What is the least possible value of $ m$?




Using a simple Combinatorial Nullstellensatz argument, we can show that if $ mu(n,k)$ is the minimum possible $ m$, then
$$ 1 + nleftlfloordfrac{k - 1}{2}rightrfloor leq mu(n,k) leq 1 + nleftlfloorleft(dfrac{k - 1}{2}right)^2rightrfloor ,.
$$

However, the lower bound is not sharp for large $ k$. All I know is that the lower bound is exact for $ k = 1$, $ 2$, $ 3$, and $ 4$, as well as when $ n = 1$, and for a special case with $ n = 2$ and $ k = 5$ (as shown in the attached figure below). While my bounds give $5leq mu(2,6) leq 13$, I believe that $mu(2,6)=6$. Can you verify this? enter image description here



Can anyone help improve these bounds? Note that the bounds also work if the $S_j$'s can be $(n-1)$-dimensional hyperellipsoids, but what will be the value of the smallest $m$ in this case (which, to avoid confusion, we shall use $nu(n,k)$ for this smallest $m$)? In other words, we also have
$$ 1 + nleftlfloordfrac{k - 1}{2}rightrfloor leq nu(n,k) leq 1 + nleftlfloorleft(dfrac{k - 1}{2}right)^2rightrfloor ,.
$$


For example, $nu(1,k)=leftlceildfrac{k+1}{2}rightrceil=mu(1,k)$ for all $kinmathbb{N}$ and $nu(2,k)=1+2leftlfloordfrac{k-1}{2}rightrfloor$ for $k=1,2,3,4,5$. Observe that the lower bound for $nu(2,6)$ is also sharp (i.e., $nu(2,6)=5$).



What happens if the $S_j$'s can be any nondegenerate $(n-1)$-dimensional hyperconic sections (where my Combinatorial Nullstellensatz argument no longer works, so the lower bound I have obtained no longer holds)? In this most general form, we shall use the notation $rho(n,k)$ for the smallest $m$. The only known bound for $rho(n,k)$ is
$$ rho(n,k) leq 1 + nleftlfloorleft(dfrac{k - 1}{2}right)^2rightrfloor ,.
$$

For example, $rho(1,k)=leftlceildfrac{k+1}{2}rightrceil=mu(1,k)$ for all $kinmathbb{N}$. However, $mu$, $nu$, and $rho$ do not always coincide. Known values are $rho(2,1)=1=rho(2,2)$ and $rho(2,3)=2<mu(2,3)$, while we have $rho(2,4)leq 3=mu(2,4)$, $rho(2,5)leq 4 <mu(2,5)$, and $rho(2,6)leq 5leqmu(2,6)$.










share|cite|improve this question











$endgroup$




Let $n,kinmathbb{N}$. In this problem, the geometry of $mathbb{R}^n$ is the usual Euclidean geometry.




The lattice hypercube $ Q(n,k)$ is defined to be the set $ {1,2,...,k}^n subseteqmathbb{R}^n$. If we want to cover this hypercube with the $m$
distinct $(n-1)$-dimensional hyperspheres $S_1$, $S_2$, $ldots$,
$S_m$. By "covering $Q(n,k)$," I mean that $Q(n,k)subseteq
bigcuplimits_{j=1}^m,S_j$
. What is the least possible value of $ m$?




Using a simple Combinatorial Nullstellensatz argument, we can show that if $ mu(n,k)$ is the minimum possible $ m$, then
$$ 1 + nleftlfloordfrac{k - 1}{2}rightrfloor leq mu(n,k) leq 1 + nleftlfloorleft(dfrac{k - 1}{2}right)^2rightrfloor ,.
$$

However, the lower bound is not sharp for large $ k$. All I know is that the lower bound is exact for $ k = 1$, $ 2$, $ 3$, and $ 4$, as well as when $ n = 1$, and for a special case with $ n = 2$ and $ k = 5$ (as shown in the attached figure below). While my bounds give $5leq mu(2,6) leq 13$, I believe that $mu(2,6)=6$. Can you verify this? enter image description here



Can anyone help improve these bounds? Note that the bounds also work if the $S_j$'s can be $(n-1)$-dimensional hyperellipsoids, but what will be the value of the smallest $m$ in this case (which, to avoid confusion, we shall use $nu(n,k)$ for this smallest $m$)? In other words, we also have
$$ 1 + nleftlfloordfrac{k - 1}{2}rightrfloor leq nu(n,k) leq 1 + nleftlfloorleft(dfrac{k - 1}{2}right)^2rightrfloor ,.
$$


For example, $nu(1,k)=leftlceildfrac{k+1}{2}rightrceil=mu(1,k)$ for all $kinmathbb{N}$ and $nu(2,k)=1+2leftlfloordfrac{k-1}{2}rightrfloor$ for $k=1,2,3,4,5$. Observe that the lower bound for $nu(2,6)$ is also sharp (i.e., $nu(2,6)=5$).



What happens if the $S_j$'s can be any nondegenerate $(n-1)$-dimensional hyperconic sections (where my Combinatorial Nullstellensatz argument no longer works, so the lower bound I have obtained no longer holds)? In this most general form, we shall use the notation $rho(n,k)$ for the smallest $m$. The only known bound for $rho(n,k)$ is
$$ rho(n,k) leq 1 + nleftlfloorleft(dfrac{k - 1}{2}right)^2rightrfloor ,.
$$

For example, $rho(1,k)=leftlceildfrac{k+1}{2}rightrceil=mu(1,k)$ for all $kinmathbb{N}$. However, $mu$, $nu$, and $rho$ do not always coincide. Known values are $rho(2,1)=1=rho(2,2)$ and $rho(2,3)=2<mu(2,3)$, while we have $rho(2,4)leq 3=mu(2,4)$, $rho(2,5)leq 4 <mu(2,5)$, and $rho(2,6)leq 5leqmu(2,6)$.







algebraic-geometry euclidean-geometry integer-lattices combinatorial-geometry extremal-combinatorics






share|cite|improve this question















share|cite|improve this question













share|cite|improve this question




share|cite|improve this question








edited Dec 13 '18 at 11:56







Batominovski

















asked Jul 24 '15 at 20:12









BatominovskiBatominovski

33.1k33293




33.1k33293












  • $begingroup$
    According to your question, 2d circles should be used in a 3d lattice hypercube. And 1d line-segments should be used in a 2d lattice hypercube. So why do you use 2d circles in the 2d lattice hypercube?
    $endgroup$
    – johannesvalks
    Jul 27 '15 at 4:47










  • $begingroup$
    The usual definition is that a unit $n$-dimensional sphere is the set $mathbb{S}^n = left{mathbf{x}inmathbb{R}^{n+1},|,|mathbf{x}|_2=1right}$, which sits in the $(n+1)$-dimensional Euclidean space. Hence, a circle is a $1$-dimensional sphere. This is consistent with the notion of manifold dimensions.
    $endgroup$
    – Batominovski
    Jul 27 '15 at 5:01












  • $begingroup$
    Ok, thanks. Now I can go to this problem again.
    $endgroup$
    – johannesvalks
    Jul 27 '15 at 5:03






  • 2




    $begingroup$
    Various papers I found that may or may not be helpful: uam.es/personal_pdi/ciencias/fchamizo/kiosco/files/lpha.pdf, link.springer.com/article/10.1007%2Fs006050050066, digital.csic.es/bitstream/10261/31107/1/… (Theorem 15), digital.csic.es/bitstream/10261/31179/1/…, uam.es/personal_pdi/ciencias/cillerue/Papers/closelattice.pdf.
    $endgroup$
    – joriki
    Jul 29 '15 at 14:41


















  • $begingroup$
    According to your question, 2d circles should be used in a 3d lattice hypercube. And 1d line-segments should be used in a 2d lattice hypercube. So why do you use 2d circles in the 2d lattice hypercube?
    $endgroup$
    – johannesvalks
    Jul 27 '15 at 4:47










  • $begingroup$
    The usual definition is that a unit $n$-dimensional sphere is the set $mathbb{S}^n = left{mathbf{x}inmathbb{R}^{n+1},|,|mathbf{x}|_2=1right}$, which sits in the $(n+1)$-dimensional Euclidean space. Hence, a circle is a $1$-dimensional sphere. This is consistent with the notion of manifold dimensions.
    $endgroup$
    – Batominovski
    Jul 27 '15 at 5:01












  • $begingroup$
    Ok, thanks. Now I can go to this problem again.
    $endgroup$
    – johannesvalks
    Jul 27 '15 at 5:03






  • 2




    $begingroup$
    Various papers I found that may or may not be helpful: uam.es/personal_pdi/ciencias/fchamizo/kiosco/files/lpha.pdf, link.springer.com/article/10.1007%2Fs006050050066, digital.csic.es/bitstream/10261/31107/1/… (Theorem 15), digital.csic.es/bitstream/10261/31179/1/…, uam.es/personal_pdi/ciencias/cillerue/Papers/closelattice.pdf.
    $endgroup$
    – joriki
    Jul 29 '15 at 14:41
















$begingroup$
According to your question, 2d circles should be used in a 3d lattice hypercube. And 1d line-segments should be used in a 2d lattice hypercube. So why do you use 2d circles in the 2d lattice hypercube?
$endgroup$
– johannesvalks
Jul 27 '15 at 4:47




$begingroup$
According to your question, 2d circles should be used in a 3d lattice hypercube. And 1d line-segments should be used in a 2d lattice hypercube. So why do you use 2d circles in the 2d lattice hypercube?
$endgroup$
– johannesvalks
Jul 27 '15 at 4:47












$begingroup$
The usual definition is that a unit $n$-dimensional sphere is the set $mathbb{S}^n = left{mathbf{x}inmathbb{R}^{n+1},|,|mathbf{x}|_2=1right}$, which sits in the $(n+1)$-dimensional Euclidean space. Hence, a circle is a $1$-dimensional sphere. This is consistent with the notion of manifold dimensions.
$endgroup$
– Batominovski
Jul 27 '15 at 5:01






$begingroup$
The usual definition is that a unit $n$-dimensional sphere is the set $mathbb{S}^n = left{mathbf{x}inmathbb{R}^{n+1},|,|mathbf{x}|_2=1right}$, which sits in the $(n+1)$-dimensional Euclidean space. Hence, a circle is a $1$-dimensional sphere. This is consistent with the notion of manifold dimensions.
$endgroup$
– Batominovski
Jul 27 '15 at 5:01














$begingroup$
Ok, thanks. Now I can go to this problem again.
$endgroup$
– johannesvalks
Jul 27 '15 at 5:03




$begingroup$
Ok, thanks. Now I can go to this problem again.
$endgroup$
– johannesvalks
Jul 27 '15 at 5:03




2




2




$begingroup$
Various papers I found that may or may not be helpful: uam.es/personal_pdi/ciencias/fchamizo/kiosco/files/lpha.pdf, link.springer.com/article/10.1007%2Fs006050050066, digital.csic.es/bitstream/10261/31107/1/… (Theorem 15), digital.csic.es/bitstream/10261/31179/1/…, uam.es/personal_pdi/ciencias/cillerue/Papers/closelattice.pdf.
$endgroup$
– joriki
Jul 29 '15 at 14:41




$begingroup$
Various papers I found that may or may not be helpful: uam.es/personal_pdi/ciencias/fchamizo/kiosco/files/lpha.pdf, link.springer.com/article/10.1007%2Fs006050050066, digital.csic.es/bitstream/10261/31107/1/… (Theorem 15), digital.csic.es/bitstream/10261/31179/1/…, uam.es/personal_pdi/ciencias/cillerue/Papers/closelattice.pdf.
$endgroup$
– joriki
Jul 29 '15 at 14:41










1 Answer
1






active

oldest

votes


















3





+100







$begingroup$

Here's a result for $n=2$.



As discussed at Bounds for the size of a circle with a fixed number of integer points, a circle of radius $R$ centred at the origin passes through at most $4(2log_5R+1)$ integer lattice points.



Our circles need not be centred at the origin, but we can get an upper bound for the denominator of the rational coordinates of their centres. Assume that at least three integer lattice points $(x_i,y_i)$ lie on a circle centred at $(X,Y)$. Then subtracting the three circle equations yields



$$
x_1^2-x_2^2+y_1^2-y_2^2=2X(x_1-x_2)+2Y(y_1-y_2);,\
x_1^2-x_3^2+y_1^2-y_3^2=2X(x_1-x_3)+2Y(y_1-y_3);.
$$



This is a linear system of equations for $X$, $Y$ with integer coefficients, and the denominator of the solution is at most $2$ times the determinant



$$
left|begin{matrix}x_1-x_2&y_1-y_2\x_1-x_3&y_1-y_3end{matrix}right|;.
$$



(Not $4$ times because one factor of $2$ is also in the determinants in the numerator.) This is the area of a parallelogram spanned by the three points, which can be at most $(k-1)^2lt k^2$, so the denominator is at most $2k^2$. We can subdivide the grid by the denominator; all lattice points will still be lattice points, and the radius of a circle of radius $r$ in terms of the new grid constant will be at most $R=2k^2r$. Thus a circle of radius $r$ can pass through at most $4(2log_5(2k^2r)+1)$ lattice points.



Now we need to bound the radius $r$. Again assume that at least three lattice points $(x_i,y_i)$ lie on the circle. Then the area of a parallelogram they span is a positive integer, and hence at least $1$. Since the distance between two of them is at most $sqrt2(k-1)$, the altitude of the third above the line connecting them must be at least $1/(sqrt2(k-1))$. For at least one of the points, this altitude intersects the opposite side. Then for given altitude, the radius of the circle is maximal if the point is symmetrically positioned between the other two points. This maximal radius is the radius of the circle that passes through $(pm (k-1),/sqrt2,0)$ and $(0,1/(sqrt2(k-1)))$, which is



$$
r_text{max}=sqrt{frac{(k-1)^2}2+frac{left((k-1)^4-1right)^2}{8(k-1)^2}}le2^{-3/2}k^3;.
$$



Plugging this into the bound derived above yields



$$
mu(2,k)gefrac{k^2}{4left(2log_5(2^{-1/2}k^5)+1right)}simfrac{log5}{40}frac{k^2}{log k}approxfrac1{25}frac{k^2}{log k};.
$$



This improves on your linear bound for $kge122$.



This could probably be improved by a more careful analysis, since circles of radius $k^3$ are very close to a line passing through the grid; at the very least the factor $4$ can asymptotically be replaced by $2$ since at most $2$ quadrants of such a large circle can intersect the lattice.





I removed some optimistic remarks about generalizing this for higher $n$. While the argument for the denominator generalizes and yields $Rle2k^nr$, and I believe the argument for the maximal radius should also generalize and again yield a bound of the order of $k^3$, what's missing to turn this into improved bounds for the present problem is a logarithmic bound on the number of lattice points on a hypersphere. In fact the MathWorld article on the sum-of-squares function gives identities that imply that such a bound doesn't hold for many $n$.



Here, in case someone can make use of them, are some results I found in the literature. This paper (for $n=3$) and this paper (for $nge4$) give bounds for the difference $Delta_n(R)$ between the number of lattice points inside an $(n-1)$-sphere of radius $R$ and its volume:



$$
begin{align}
Delta_n(R)in Oleft(R^thetaright)quadtext{with}quadtheta=
begin{cases}
frac{21}{16}+epsilon&n=3\
2+epsilon&n=4\
n-2&nge5
end{cases}
end{align}
$$



Since the number of lattice points in the sphere jumps by the number of lattice points on the sphere, the latter must satisfy the same bounds.






share|cite|improve this answer











$endgroup$









  • 1




    $begingroup$
    Despite the incomplete answer, I think your reply gives me quite a lot of insight to this problem. Thanks.
    $endgroup$
    – Batominovski
    Aug 3 '15 at 7:32











Your Answer





StackExchange.ifUsing("editor", function () {
return StackExchange.using("mathjaxEditing", function () {
StackExchange.MarkdownEditor.creationCallbacks.add(function (editor, postfix) {
StackExchange.mathjaxEditing.prepareWmdForMathJax(editor, postfix, [["$", "$"], ["\\(","\\)"]]);
});
});
}, "mathjax-editing");

StackExchange.ready(function() {
var channelOptions = {
tags: "".split(" "),
id: "69"
};
initTagRenderer("".split(" "), "".split(" "), channelOptions);

StackExchange.using("externalEditor", function() {
// Have to fire editor after snippets, if snippets enabled
if (StackExchange.settings.snippets.snippetsEnabled) {
StackExchange.using("snippets", function() {
createEditor();
});
}
else {
createEditor();
}
});

function createEditor() {
StackExchange.prepareEditor({
heartbeatType: 'answer',
autoActivateHeartbeat: false,
convertImagesToLinks: true,
noModals: true,
showLowRepImageUploadWarning: true,
reputationToPostImages: 10,
bindNavPrevention: true,
postfix: "",
imageUploader: {
brandingHtml: "Powered by u003ca class="icon-imgur-white" href="https://imgur.com/"u003eu003c/au003e",
contentPolicyHtml: "User contributions licensed under u003ca href="https://creativecommons.org/licenses/by-sa/3.0/"u003ecc by-sa 3.0 with attribution requiredu003c/au003e u003ca href="https://stackoverflow.com/legal/content-policy"u003e(content policy)u003c/au003e",
allowUrls: true
},
noCode: true, onDemand: true,
discardSelector: ".discard-answer"
,immediatelyShowMarkdownHelp:true
});


}
});














draft saved

draft discarded


















StackExchange.ready(
function () {
StackExchange.openid.initPostLogin('.new-post-login', 'https%3a%2f%2fmath.stackexchange.com%2fquestions%2f1372869%2fhypercube-and-hyperspheres%23new-answer', 'question_page');
}
);

Post as a guest















Required, but never shown

























1 Answer
1






active

oldest

votes








1 Answer
1






active

oldest

votes









active

oldest

votes






active

oldest

votes









3





+100







$begingroup$

Here's a result for $n=2$.



As discussed at Bounds for the size of a circle with a fixed number of integer points, a circle of radius $R$ centred at the origin passes through at most $4(2log_5R+1)$ integer lattice points.



Our circles need not be centred at the origin, but we can get an upper bound for the denominator of the rational coordinates of their centres. Assume that at least three integer lattice points $(x_i,y_i)$ lie on a circle centred at $(X,Y)$. Then subtracting the three circle equations yields



$$
x_1^2-x_2^2+y_1^2-y_2^2=2X(x_1-x_2)+2Y(y_1-y_2);,\
x_1^2-x_3^2+y_1^2-y_3^2=2X(x_1-x_3)+2Y(y_1-y_3);.
$$



This is a linear system of equations for $X$, $Y$ with integer coefficients, and the denominator of the solution is at most $2$ times the determinant



$$
left|begin{matrix}x_1-x_2&y_1-y_2\x_1-x_3&y_1-y_3end{matrix}right|;.
$$



(Not $4$ times because one factor of $2$ is also in the determinants in the numerator.) This is the area of a parallelogram spanned by the three points, which can be at most $(k-1)^2lt k^2$, so the denominator is at most $2k^2$. We can subdivide the grid by the denominator; all lattice points will still be lattice points, and the radius of a circle of radius $r$ in terms of the new grid constant will be at most $R=2k^2r$. Thus a circle of radius $r$ can pass through at most $4(2log_5(2k^2r)+1)$ lattice points.



Now we need to bound the radius $r$. Again assume that at least three lattice points $(x_i,y_i)$ lie on the circle. Then the area of a parallelogram they span is a positive integer, and hence at least $1$. Since the distance between two of them is at most $sqrt2(k-1)$, the altitude of the third above the line connecting them must be at least $1/(sqrt2(k-1))$. For at least one of the points, this altitude intersects the opposite side. Then for given altitude, the radius of the circle is maximal if the point is symmetrically positioned between the other two points. This maximal radius is the radius of the circle that passes through $(pm (k-1),/sqrt2,0)$ and $(0,1/(sqrt2(k-1)))$, which is



$$
r_text{max}=sqrt{frac{(k-1)^2}2+frac{left((k-1)^4-1right)^2}{8(k-1)^2}}le2^{-3/2}k^3;.
$$



Plugging this into the bound derived above yields



$$
mu(2,k)gefrac{k^2}{4left(2log_5(2^{-1/2}k^5)+1right)}simfrac{log5}{40}frac{k^2}{log k}approxfrac1{25}frac{k^2}{log k};.
$$



This improves on your linear bound for $kge122$.



This could probably be improved by a more careful analysis, since circles of radius $k^3$ are very close to a line passing through the grid; at the very least the factor $4$ can asymptotically be replaced by $2$ since at most $2$ quadrants of such a large circle can intersect the lattice.





I removed some optimistic remarks about generalizing this for higher $n$. While the argument for the denominator generalizes and yields $Rle2k^nr$, and I believe the argument for the maximal radius should also generalize and again yield a bound of the order of $k^3$, what's missing to turn this into improved bounds for the present problem is a logarithmic bound on the number of lattice points on a hypersphere. In fact the MathWorld article on the sum-of-squares function gives identities that imply that such a bound doesn't hold for many $n$.



Here, in case someone can make use of them, are some results I found in the literature. This paper (for $n=3$) and this paper (for $nge4$) give bounds for the difference $Delta_n(R)$ between the number of lattice points inside an $(n-1)$-sphere of radius $R$ and its volume:



$$
begin{align}
Delta_n(R)in Oleft(R^thetaright)quadtext{with}quadtheta=
begin{cases}
frac{21}{16}+epsilon&n=3\
2+epsilon&n=4\
n-2&nge5
end{cases}
end{align}
$$



Since the number of lattice points in the sphere jumps by the number of lattice points on the sphere, the latter must satisfy the same bounds.






share|cite|improve this answer











$endgroup$









  • 1




    $begingroup$
    Despite the incomplete answer, I think your reply gives me quite a lot of insight to this problem. Thanks.
    $endgroup$
    – Batominovski
    Aug 3 '15 at 7:32
















3





+100







$begingroup$

Here's a result for $n=2$.



As discussed at Bounds for the size of a circle with a fixed number of integer points, a circle of radius $R$ centred at the origin passes through at most $4(2log_5R+1)$ integer lattice points.



Our circles need not be centred at the origin, but we can get an upper bound for the denominator of the rational coordinates of their centres. Assume that at least three integer lattice points $(x_i,y_i)$ lie on a circle centred at $(X,Y)$. Then subtracting the three circle equations yields



$$
x_1^2-x_2^2+y_1^2-y_2^2=2X(x_1-x_2)+2Y(y_1-y_2);,\
x_1^2-x_3^2+y_1^2-y_3^2=2X(x_1-x_3)+2Y(y_1-y_3);.
$$



This is a linear system of equations for $X$, $Y$ with integer coefficients, and the denominator of the solution is at most $2$ times the determinant



$$
left|begin{matrix}x_1-x_2&y_1-y_2\x_1-x_3&y_1-y_3end{matrix}right|;.
$$



(Not $4$ times because one factor of $2$ is also in the determinants in the numerator.) This is the area of a parallelogram spanned by the three points, which can be at most $(k-1)^2lt k^2$, so the denominator is at most $2k^2$. We can subdivide the grid by the denominator; all lattice points will still be lattice points, and the radius of a circle of radius $r$ in terms of the new grid constant will be at most $R=2k^2r$. Thus a circle of radius $r$ can pass through at most $4(2log_5(2k^2r)+1)$ lattice points.



Now we need to bound the radius $r$. Again assume that at least three lattice points $(x_i,y_i)$ lie on the circle. Then the area of a parallelogram they span is a positive integer, and hence at least $1$. Since the distance between two of them is at most $sqrt2(k-1)$, the altitude of the third above the line connecting them must be at least $1/(sqrt2(k-1))$. For at least one of the points, this altitude intersects the opposite side. Then for given altitude, the radius of the circle is maximal if the point is symmetrically positioned between the other two points. This maximal radius is the radius of the circle that passes through $(pm (k-1),/sqrt2,0)$ and $(0,1/(sqrt2(k-1)))$, which is



$$
r_text{max}=sqrt{frac{(k-1)^2}2+frac{left((k-1)^4-1right)^2}{8(k-1)^2}}le2^{-3/2}k^3;.
$$



Plugging this into the bound derived above yields



$$
mu(2,k)gefrac{k^2}{4left(2log_5(2^{-1/2}k^5)+1right)}simfrac{log5}{40}frac{k^2}{log k}approxfrac1{25}frac{k^2}{log k};.
$$



This improves on your linear bound for $kge122$.



This could probably be improved by a more careful analysis, since circles of radius $k^3$ are very close to a line passing through the grid; at the very least the factor $4$ can asymptotically be replaced by $2$ since at most $2$ quadrants of such a large circle can intersect the lattice.





I removed some optimistic remarks about generalizing this for higher $n$. While the argument for the denominator generalizes and yields $Rle2k^nr$, and I believe the argument for the maximal radius should also generalize and again yield a bound of the order of $k^3$, what's missing to turn this into improved bounds for the present problem is a logarithmic bound on the number of lattice points on a hypersphere. In fact the MathWorld article on the sum-of-squares function gives identities that imply that such a bound doesn't hold for many $n$.



Here, in case someone can make use of them, are some results I found in the literature. This paper (for $n=3$) and this paper (for $nge4$) give bounds for the difference $Delta_n(R)$ between the number of lattice points inside an $(n-1)$-sphere of radius $R$ and its volume:



$$
begin{align}
Delta_n(R)in Oleft(R^thetaright)quadtext{with}quadtheta=
begin{cases}
frac{21}{16}+epsilon&n=3\
2+epsilon&n=4\
n-2&nge5
end{cases}
end{align}
$$



Since the number of lattice points in the sphere jumps by the number of lattice points on the sphere, the latter must satisfy the same bounds.






share|cite|improve this answer











$endgroup$









  • 1




    $begingroup$
    Despite the incomplete answer, I think your reply gives me quite a lot of insight to this problem. Thanks.
    $endgroup$
    – Batominovski
    Aug 3 '15 at 7:32














3





+100







3





+100



3




+100



$begingroup$

Here's a result for $n=2$.



As discussed at Bounds for the size of a circle with a fixed number of integer points, a circle of radius $R$ centred at the origin passes through at most $4(2log_5R+1)$ integer lattice points.



Our circles need not be centred at the origin, but we can get an upper bound for the denominator of the rational coordinates of their centres. Assume that at least three integer lattice points $(x_i,y_i)$ lie on a circle centred at $(X,Y)$. Then subtracting the three circle equations yields



$$
x_1^2-x_2^2+y_1^2-y_2^2=2X(x_1-x_2)+2Y(y_1-y_2);,\
x_1^2-x_3^2+y_1^2-y_3^2=2X(x_1-x_3)+2Y(y_1-y_3);.
$$



This is a linear system of equations for $X$, $Y$ with integer coefficients, and the denominator of the solution is at most $2$ times the determinant



$$
left|begin{matrix}x_1-x_2&y_1-y_2\x_1-x_3&y_1-y_3end{matrix}right|;.
$$



(Not $4$ times because one factor of $2$ is also in the determinants in the numerator.) This is the area of a parallelogram spanned by the three points, which can be at most $(k-1)^2lt k^2$, so the denominator is at most $2k^2$. We can subdivide the grid by the denominator; all lattice points will still be lattice points, and the radius of a circle of radius $r$ in terms of the new grid constant will be at most $R=2k^2r$. Thus a circle of radius $r$ can pass through at most $4(2log_5(2k^2r)+1)$ lattice points.



Now we need to bound the radius $r$. Again assume that at least three lattice points $(x_i,y_i)$ lie on the circle. Then the area of a parallelogram they span is a positive integer, and hence at least $1$. Since the distance between two of them is at most $sqrt2(k-1)$, the altitude of the third above the line connecting them must be at least $1/(sqrt2(k-1))$. For at least one of the points, this altitude intersects the opposite side. Then for given altitude, the radius of the circle is maximal if the point is symmetrically positioned between the other two points. This maximal radius is the radius of the circle that passes through $(pm (k-1),/sqrt2,0)$ and $(0,1/(sqrt2(k-1)))$, which is



$$
r_text{max}=sqrt{frac{(k-1)^2}2+frac{left((k-1)^4-1right)^2}{8(k-1)^2}}le2^{-3/2}k^3;.
$$



Plugging this into the bound derived above yields



$$
mu(2,k)gefrac{k^2}{4left(2log_5(2^{-1/2}k^5)+1right)}simfrac{log5}{40}frac{k^2}{log k}approxfrac1{25}frac{k^2}{log k};.
$$



This improves on your linear bound for $kge122$.



This could probably be improved by a more careful analysis, since circles of radius $k^3$ are very close to a line passing through the grid; at the very least the factor $4$ can asymptotically be replaced by $2$ since at most $2$ quadrants of such a large circle can intersect the lattice.





I removed some optimistic remarks about generalizing this for higher $n$. While the argument for the denominator generalizes and yields $Rle2k^nr$, and I believe the argument for the maximal radius should also generalize and again yield a bound of the order of $k^3$, what's missing to turn this into improved bounds for the present problem is a logarithmic bound on the number of lattice points on a hypersphere. In fact the MathWorld article on the sum-of-squares function gives identities that imply that such a bound doesn't hold for many $n$.



Here, in case someone can make use of them, are some results I found in the literature. This paper (for $n=3$) and this paper (for $nge4$) give bounds for the difference $Delta_n(R)$ between the number of lattice points inside an $(n-1)$-sphere of radius $R$ and its volume:



$$
begin{align}
Delta_n(R)in Oleft(R^thetaright)quadtext{with}quadtheta=
begin{cases}
frac{21}{16}+epsilon&n=3\
2+epsilon&n=4\
n-2&nge5
end{cases}
end{align}
$$



Since the number of lattice points in the sphere jumps by the number of lattice points on the sphere, the latter must satisfy the same bounds.






share|cite|improve this answer











$endgroup$



Here's a result for $n=2$.



As discussed at Bounds for the size of a circle with a fixed number of integer points, a circle of radius $R$ centred at the origin passes through at most $4(2log_5R+1)$ integer lattice points.



Our circles need not be centred at the origin, but we can get an upper bound for the denominator of the rational coordinates of their centres. Assume that at least three integer lattice points $(x_i,y_i)$ lie on a circle centred at $(X,Y)$. Then subtracting the three circle equations yields



$$
x_1^2-x_2^2+y_1^2-y_2^2=2X(x_1-x_2)+2Y(y_1-y_2);,\
x_1^2-x_3^2+y_1^2-y_3^2=2X(x_1-x_3)+2Y(y_1-y_3);.
$$



This is a linear system of equations for $X$, $Y$ with integer coefficients, and the denominator of the solution is at most $2$ times the determinant



$$
left|begin{matrix}x_1-x_2&y_1-y_2\x_1-x_3&y_1-y_3end{matrix}right|;.
$$



(Not $4$ times because one factor of $2$ is also in the determinants in the numerator.) This is the area of a parallelogram spanned by the three points, which can be at most $(k-1)^2lt k^2$, so the denominator is at most $2k^2$. We can subdivide the grid by the denominator; all lattice points will still be lattice points, and the radius of a circle of radius $r$ in terms of the new grid constant will be at most $R=2k^2r$. Thus a circle of radius $r$ can pass through at most $4(2log_5(2k^2r)+1)$ lattice points.



Now we need to bound the radius $r$. Again assume that at least three lattice points $(x_i,y_i)$ lie on the circle. Then the area of a parallelogram they span is a positive integer, and hence at least $1$. Since the distance between two of them is at most $sqrt2(k-1)$, the altitude of the third above the line connecting them must be at least $1/(sqrt2(k-1))$. For at least one of the points, this altitude intersects the opposite side. Then for given altitude, the radius of the circle is maximal if the point is symmetrically positioned between the other two points. This maximal radius is the radius of the circle that passes through $(pm (k-1),/sqrt2,0)$ and $(0,1/(sqrt2(k-1)))$, which is



$$
r_text{max}=sqrt{frac{(k-1)^2}2+frac{left((k-1)^4-1right)^2}{8(k-1)^2}}le2^{-3/2}k^3;.
$$



Plugging this into the bound derived above yields



$$
mu(2,k)gefrac{k^2}{4left(2log_5(2^{-1/2}k^5)+1right)}simfrac{log5}{40}frac{k^2}{log k}approxfrac1{25}frac{k^2}{log k};.
$$



This improves on your linear bound for $kge122$.



This could probably be improved by a more careful analysis, since circles of radius $k^3$ are very close to a line passing through the grid; at the very least the factor $4$ can asymptotically be replaced by $2$ since at most $2$ quadrants of such a large circle can intersect the lattice.





I removed some optimistic remarks about generalizing this for higher $n$. While the argument for the denominator generalizes and yields $Rle2k^nr$, and I believe the argument for the maximal radius should also generalize and again yield a bound of the order of $k^3$, what's missing to turn this into improved bounds for the present problem is a logarithmic bound on the number of lattice points on a hypersphere. In fact the MathWorld article on the sum-of-squares function gives identities that imply that such a bound doesn't hold for many $n$.



Here, in case someone can make use of them, are some results I found in the literature. This paper (for $n=3$) and this paper (for $nge4$) give bounds for the difference $Delta_n(R)$ between the number of lattice points inside an $(n-1)$-sphere of radius $R$ and its volume:



$$
begin{align}
Delta_n(R)in Oleft(R^thetaright)quadtext{with}quadtheta=
begin{cases}
frac{21}{16}+epsilon&n=3\
2+epsilon&n=4\
n-2&nge5
end{cases}
end{align}
$$



Since the number of lattice points in the sphere jumps by the number of lattice points on the sphere, the latter must satisfy the same bounds.







share|cite|improve this answer














share|cite|improve this answer



share|cite|improve this answer








edited Apr 13 '17 at 12:21









Community

1




1










answered Jul 29 '15 at 8:31









jorikijoriki

171k10188349




171k10188349








  • 1




    $begingroup$
    Despite the incomplete answer, I think your reply gives me quite a lot of insight to this problem. Thanks.
    $endgroup$
    – Batominovski
    Aug 3 '15 at 7:32














  • 1




    $begingroup$
    Despite the incomplete answer, I think your reply gives me quite a lot of insight to this problem. Thanks.
    $endgroup$
    – Batominovski
    Aug 3 '15 at 7:32








1




1




$begingroup$
Despite the incomplete answer, I think your reply gives me quite a lot of insight to this problem. Thanks.
$endgroup$
– Batominovski
Aug 3 '15 at 7:32




$begingroup$
Despite the incomplete answer, I think your reply gives me quite a lot of insight to this problem. Thanks.
$endgroup$
– Batominovski
Aug 3 '15 at 7:32


















draft saved

draft discarded




















































Thanks for contributing an answer to Mathematics Stack Exchange!


  • Please be sure to answer the question. Provide details and share your research!

But avoid



  • Asking for help, clarification, or responding to other answers.

  • Making statements based on opinion; back them up with references or personal experience.


Use MathJax to format equations. MathJax reference.


To learn more, see our tips on writing great answers.




draft saved


draft discarded














StackExchange.ready(
function () {
StackExchange.openid.initPostLogin('.new-post-login', 'https%3a%2f%2fmath.stackexchange.com%2fquestions%2f1372869%2fhypercube-and-hyperspheres%23new-answer', 'question_page');
}
);

Post as a guest















Required, but never shown





















































Required, but never shown














Required, but never shown












Required, but never shown







Required, but never shown

































Required, but never shown














Required, but never shown












Required, but never shown







Required, but never shown







Popular posts from this blog

Plaza Victoria

In PowerPoint, is there a keyboard shortcut for bulleted / numbered list?

How to put 3 figures in Latex with 2 figures side by side and 1 below these side by side images but in...